There are three bells that ring at different intervals. The first bell rings every 6 minutes, the second
bell rings every 7 minutes, The third bell rings 9 minutes. If all the bells ring together at noon, at what
time will they next all ring together?

Answers

Answer 1

Answer:

2:06

Step-by-step explanation:

You just need to find the Least Common Multiple of the 3 numbers. In this case it was 126 which if those are minutes that would mean 2 hours and 6 minutes.


Related Questions

the ratio of peanuts to raisins in a snack mix is 5:4. A bag of the snack mix contains 200 peanuts. How many raisins are in the bag?

Answers

Number of raisins in the bag is 160 raisins

Given that;

Ratio of peanuts to raisins = 5:4

Bag of the snack mix contains = 200 peanuts

Find:

Number of raisins in the bag

Computation:

Number of raisins in the bag = 4[200/5]

Number of raisins in the bag = 4[40]

Number of raisins in the bag = 160 raisins

Learn more:

https://brainly.com/question/1383782?referrer=searchResults

Sketch the curve represented by the parametric equations (indicate the orientation of the curve), and write the corresponding rectangular equation by eliminating the parameter.
x = 5t − 2, y = 2t + 1.

Answers

Please find attached

Answer:

equation : 2x+9-5y=0

Step-by-step explanation:

x and y cut axes at -4.5 and 1.4 respectively

given

when x=0

9-5y=0

y=1.4

When y=0

2x+9=0

x=-4.5

The scores of players on a golf team are shown on the table Celia-3 Janine 3 Sami 1 Ted 4 Travis 0 The team combined score was 0. What was Travis score?

Answers

His score would have to be -5:

-3 + 3 = 0

0 + 1 = 1

1 + 4 = 5

5 + (-5) = 0

So you know that the su of all the scores is 0.

The scores in the table are cecilia -3, janine 3, sami 1 and ted 4 and travis x, where x is the number that we want to know.

if we sum those, we have : 3 - 3 + 4 + 1 + x = 0

and we get 5 + x = 0

then x= -5, so travis's score is -5 points.

Mrs. Phillips ordered 56 sub sandwiches for a team meal, and 2/7 of them were ham. how many of the sandwiches were ham?​

Answers

Answer:

16

Step-by-step explanation:

Take the total number of sandwiches and multiply the fraction that were ham

56 * 2/7

56/7 *2

8 *2

16

16 sandwiches were ham

You want to buy a car. The loan amount will be $18,000. The company is offering a 5% interest rate for 48 months (4 years). What will your monthly payments be?

Answers

Answer:

393.5$

Step-by-step explanation:

$18,000 -- car price

Total sum (105%) = [tex]\frac{18000}{100}[/tex]* 105%

Hence

[tex]\frac{18000/100*105}{48}[/tex] = 393.5$

: What is the volume of this triangular right prism?
6.5 ft
6 ft-
11 ft
5 ft

Answers

Answer:

[tex] Volume_{Prism} = 165 ft^3 [/tex]

Step-by-step explanation:

Volume of triangular prism is given as base area * height of prism

Base area = area of triangle = ½*base of the traingle*height of the traingle

Base area = 5 ft * 6 ft = 30 ft²

Height of prism = 11 ft

Volume of the prism = [tex] \frac{1}{2}*30*11 [/tex]

[tex] Volume_{Prism} = 15*11 [/tex]

[tex] Volume_{Prism} = 165 ft^3 [/tex]

What is the solution for this inequality?

-10x < 40

Answers

Answer:

Step-by-step explanation:

-10x < 40

x > -4

can someone explain the quadratic formula to me.

Answers

Answer:

[See Below]

Step-by-step explanation:

Here are some pictures to help you.

The quadratic formula is shown below.

The beauty of the quadratic formula is that as long as your quadraticis in the form ax² + bx + c = 0, where a, b, and c are integers, you can go straight to the answer by simply plugging your values for a, b, and c into the quadratic formula.

It's a good idea to memorize it.

Believe me it's worth it, you'll be using it all the time.

PLEASE ANSWER QUICK!Kierran is trying to find the circumference of a circle. He knows that the radius (r) of the circle is 6 feet and that the diameter, d=2r. Which formula should he use?
A) A=3.14(2r)^2
B) C=3.14(2r)
C) A=3.14d^3
D) C=3.14(2D)

Answers

Answer:

B) C=3.14(2r)

Step-by-step explanation:

Circumference of a circle= 2πr

radius=6 feet

Diameter= 2r

=2*6

=12 feet

π= 3.14

C= 2πr

= 2 * 3.14 * r

= 3.14 * 2 * r

C= 3.14 (2r)

Kierran should use the formula C= 3.14 (2r) to find the circumstance of the circle

Option B is the correct answer

C= 3.14 (2r)

= 3.14 * 2 * 6

= 37.68 eet

Solve:
A rectangular garden has length of 24m and width of 17.2m. Use the correct number of
significant digits to write the perimeter of the garden.
meters

Answers

Answer:

82.4

Step-by-step explanation:

how do you a shade a venn diagram of B-A

Answers

Don't shade the part A and the part between A and B. Shade the B part only.

Graph the parabola y=x^2+3 by plotting any three points on the parabola. Move the key points on the graph to create the parabola.

Answers

Answer:

Use a graphing calc.

Step-by-step explanation:

If graphing by hand, use the vertex as one of the main points and then find other points by plugging in x-values.

Answer:

See be'ow

Step-by-step explanation:

We can use a graphing calculator to plot the points of the parabola.

See the attached file!


A jug has a maximum capacity of 4 liters. It's filled to 85% of its capacity, and then poured out at a rate of 25 ml/s. Which of the following
expressions models the volume remaining in the jug, in liters, after x seconds?

Answers

Answer: W(x) = 3.4 L - (25/1000)(L/s)*x

Step-by-step explanation:

The maximum of the jug is 4 L.

now, the jug is 85% filled, then the amount of water in the jug is:

(85%/100%)*4L = 0.85*4L = 3.4L

Now, we pour 25 mL each second, so we could write a linear equation as:

W(x) = 3.4L - (25ml/s)*x

where x is the number of seconds that passed since the beginning,

But we want to write this in Liters, we have that:

1L = 1000mL.

Then 1mL = (1/1000) L

then we can write 25 mL as:

25m L = 25*(1/1000) L = (25/1000) L.

Then we can write the equation as

W(x) = 3.4 L - (25/1000)(L/s)*x

which is the amount of water remaining in the jug after x seconds.

The West Edmonton Mall in Canada is the largest mall in the world. 20,000,000 people visit the mall each year. If the mall is opened every day, what is the average number of people per day at the mall? (1 yr. – 365 days)​

Answers

Answer:

54,795

Step-by-step explanation:

Donald has 456 marbles. Joe has 987 marbles. How many more marbles does Joe have?

Answers

Answer:

531

Start off by subtracting.

Joe = 987 marbles

Donald = 456 marbles.

987 - 456 = 531.

So that is your answer. Hope it helps!

Answer:

agree with previos answer

Step-by-step explanation:

Determine the value of c so that f(x) is continuous on the entire real line when
f(x) = {x + 3 x less than or equal to -1 2x - c x > -1.
a. -4.
b. 4.
c. 0.
d. -1.
e. none of these.

Answers

Answer:

A. -4

Step-by-step explanation:

Given the function f(x) = x + 3 for x ≤ -1 and 2x - c for x > -1, for the function to be continuous, the right hand limit of the function must be equal to its left hand limit.

For the left hand limit;

The function at the left hand occurs at x<-1

f-(x) = x+3

f-(-1) = -1+3

f-(-1) = 2

For the right hand limit, the function occurs at x>-1

f+(x) = 2x-c

f+(-1) = 2(-1)-c

f+(-1) = -2-c

For the function f(x) to be continuous on the entire real line at x = -1, then

f-(-1) = f+(-1)

On equating both sides:

2 = -2-c

Add 2 to both sides

2+2 = -2-c+2

4 =-c

Multiply both sides by minus.

-(-c) = -4

c = -4

Hence the value of c so that f(x) is continuous on the entire real line is -4

The value of 'c' is -4 and this can be determined by using the concept of continuous function and arithmetic operations.

Given :

f(x) is continuous on the entire real line when c f(x) = x + 3 for [tex]x \leq -1[/tex], 2x - c for x > -1.

Remember for a continuous function, the left-hand limit is equal to the right-hand limit. So, determine the left-hand and right-hand limit.

The left-hand limit is calculated as:

f(-1) = (-1) + 3

f(-1) = 2    --- (1)

The right-hand limit is calculated as:

f(-1) = 2(-1) - c

f(-1) = -2 - c   --- (2)

Now, equate both the expression (1) and (2).

2 = -2 - c

c = -4

For more information, refer to the link given below:

https://brainly.com/question/1111011

find the probability of Rolling a number greater than or equal to 4 on a standard die

Answers

Answer:

P(E)=1/2

Step-by-step explanation:

Total numbers=6

Numbers greater than or equal to 4= 3(4,5,6)

P(E)= favourable outcomes/total outcomes

P(E)=3/6

     =1/2

Answer:

4/6

Step-by-step explanation:

probability of rolling a num greater than or equal to 4 = 4/6

=2/3

Is 10 greater than 10.01 or less than​

Answers

Step-by-step explanation:

10 is less than 10.01

10.01-10=0.01

10 is 0.01 less than 10.01

So, 10 is less than 10.01.

Hope it helps you!!

Answer: 10 is less than 10.01

Explanation:

10.00 < 10.01

Because .00 is less than .01

find sin60 A.1/2 B.2/2 C.3/2

Answers

Answer: [tex]\frac{\sqrt{3} }{2}[/tex]

Step-by-step explanation:

To find sin(60), you need to know the unit circle. Sin(60) is the same as sin(π/3). On the unit circle, sin(π/3) is √3/2. We know this because the coordinates are (cos, sin). Sin is the y coordinate.

Answer:

[tex]\huge\boxed{\sin60^o=\dfrac{\sqrt3}{2}}[/tex]

Step-by-step explanation:

Let's take a right triangle with acute angles 30° and 60°.

We know that the ratio of the sides of such a triangle is 2 : 1 : √3

(look at the picture)

We know:

[tex]sine=\dfrac{opposite}{hypotenuse}[/tex]

In this triangle we have:

[tex]opposite=a\sqrt3\\hypotenuse=2a[/tex]

Substitute:

[tex]\sin60^o=\dfrac{a\sqrt3}{2a}[/tex]         cancel a

[tex]\sin60^o=\dfrac{\sqrt3}{2}[/tex]

Jan spent $99.99 for a comforter set, $12.99 for pillows, and $36.98 for towels. How much total did she
spend?
O $136.97
O $50.06
O $149.96
O $112.94
O $155.96

Answers

Answer:

$149.96

Step-by-step explanation:

99.99+12.99+36.98=149.96

Total=$149.96

The answer to the problem is the third option, 149.96. What you are doing is adding all of the numbers that are given together to create the total.

Endpoints of segment MN have coordinates (0, 0), (5, 1). The endpoints of segment AB have coordinates (1 1/2 , 2 1/4 ) and (−2 1/4 , k). What value of k makes these segments parallel?

Answers

Answer:

k=21

Step-by-step explanation:

Find the slope of MN which is 1/5, (use the slope formula.)

Use the slope formula for AB using k as y_2

Substitute and you get k=21

21-2.25/-2.25-1.5=-5

The lines are perpendicular because the slopes are the negative reciprocal of one another.

For K = 5.3, the line segments are parallel.

We have endpoints of segment MN have coordinates (0, 0), (5, 1) and  endpoints of segment AB have coordinates (1 1/2 , 2 1/4 ) and (−2 1/4 , k).

We have to determine the value of k that makes these segments parallel.

What is the condition for the lines to be parallel?

For lines to be parallel - slope of line 1 should be equal to slope of line 2.

m(1) = m(2)

According to question, we have -

Coordinates of M = (0, 0)

Coordinates of N = (5, 1)

Therefore, the slope of line MN = [tex]\frac{1 -0}{5-0}[/tex] = 1/5

Coordinates of A = (11/2, 21/4)

Coordinates of B = (-21/4, k)

Therefore, the slope of line AB =

[tex]$\frac{k -\frac{21}{4} }{\frac{-21}{4} -\frac{11}{2} }[/tex]  =  [tex]$\frac{k - \frac{21}{4} }{\frac{1}{4} }[/tex]  =  [tex]$\frac{\frac{4k - 21}{4} }{\frac{1}{4} }[/tex]  = 4k -21

Now, for the lines to be parallel -

m(MN) = M(AB)

4k - 21 = [tex]\frac{1}{5}[/tex]

5(4k - 21) = 1

20k - 105 = 1

20k = 106

k = 5.3

Hence, for K = 5.3, the line segments are parallel.

To solve more questions on parallel lines, visit the link below -

https://brainly.com/question/13311065

#SPJ2

if you record a lower temperature than the actual value, how would that affect the calculated value of R

Answers

Answer:

the calculated value of R would be incorrect and it would be greater than the actual value

Step-by-step explanation:

From the general gas equation,

We have a mole of gas as

PV =nRT

We will have to make R the subject of the formula

So we divide through by nT

R = PV/nT

So we have an inverse proportion between R and absolute temperature.

So when we record a lower temperature than what the actual temperature should be, R is going to be affected such that the wrongly calculated value of R would be more than the actual value.

HELP WOULD BE GREATLY APPRECIATED GURANTED BRAINLIEST

Answers

Answer: Choice A) 9

=================================================

Explanation:

We have some variable r that we divide over 3, and then we add on 5. Note the order of operations PEMDAS says to divide first and then add (since D comes before A in PEMDAS).

When we isolate r, we follow PEMDAS in reverse and undo everything. We undo addition by subtracting 5 from both sides

[tex]\frac{r}{3} + 5 \le 8\\\\\frac{r}{3} + 5-5 \le 8-5\\\\\frac{r}{3} \le 3\\\\[/tex]

Then we undo the division happening to r. So we multiply both sides by 3

[tex]\frac{r}{3} \le 3\\\\3*\frac{r}{3} \le 3*3\\\\r \le 9\\\\[/tex]

Since r is less than or equal to 9, this means that r could be 9 or smaller

Only choice A applies.

Any system of government based on rule by the people is called

Answers

Answer:

a democracy

I hope this helps!

Prove the following using a direct proof. Your proof should be expressed in com-plete English sentences.
If a, b, and c are integers such that b is a multiple of a3 and c is a multiple of b2, then c is a multiple of a6.

Answers

Answer:

Proved

Step-by-step explanation:

Given

Integers: a, b, c

Required

Show that c is a multiple of a⁶

First, we need to list out the multiples of a³

[tex]a^3 -> a^3, a^6, a^9, a^{12}...[/tex]

Since b is a multiple of a³, then b can be any of the listed multiples

Next, is to list out the multiples of b²

[tex]b^2 -> b^2,b^4,b^6,b^8,b^{10}....[/tex]

Since c is a multiple of b², then c can be any of the listed multiples

To list out the multiples of a⁶, we have to get the common multiples of a³ and b² in terms of a

Substitute a³ for b in

[tex]b^2 -> b^2,b^4,b^6,b^8,b^{10}....[/tex]

[tex](a^3)^2 -> (a^3)^2,(a^3)^4,(a^3)^6,(a^3)^8,(a^3)^{10}....[/tex]

[tex]a^6 -> a^6,a^{12},a^{18},a^{24},a^{30}....[/tex]

Recall that c is a multiple of b²

From the above listed multiples, we have a⁶ listed as one of the multiple; Hence, c is a multiple of a

Yes C should be multiple of [tex]a^6[/tex]

Calculation of the multiple:

Since it is mentioned that b should be the multiple of [tex]a^3[/tex]

So,

[tex]b = a^3R[/tex] here some integer should be R

And, the c should be multiple of [tex]b^2[/tex]

So,

[tex]C = b^2 r[/tex] for some integer r

Now we have to substitute the value of b from the first equation

i.e.

[tex]C = (a^3R)^2r = a^6 (R^2r)[/tex]

Here [tex]R^2r[/tex] is some integer

so,

[tex]C = a^6R'[/tex]

Therefore, we can say that Yes C should be multiple of [tex]a^6[/tex]

Learn more about multiple here: https://brainly.com/question/24146205

dwight can drive 60 miles for each gallon of gas on his motorcycle how far can dwight go if he has 3 gallons of gas

Answers

Answer:

180

Step-by-step explanation: 60+60 =120 120+6

According to Newton's Second Law of Motion, the sum of the forces that act on an object with a mass mmm that moves in an acceleration aaa is equal to m\cdot am⋅am, dot, a. An object whose mass in 808080 grams has an acceleration of 202020 meters per seconds square. What calculation will give us the sum of the forces that act on the object, in Newtons (which are \dfrac{\text{kg}\cdot\text{m}}{\text{s}^2} s 2 kg⋅m ​ start fraction, start text, k, g, end text, dot, start text, m, end text, divided by, start text, s, end text, squared, end fraction)?
What calculation will give us the sum of the forces that act on the object, in Newtons (which are \dfrac{\text{kg}\cdot\text{m}}{\text{s}^2}

[tex]kg . m \ S^{2}[/tex]

Answers

Answer:

If you are on Khan Academy doing this, then the answer is 80/100 X 20

                                                                                                 

Step-by-step explanation:

If the mass of object is 808080 grams and acceleration is 202020 meter per seconds square then the force will be 1632483216 kg [tex]m^{2}[/tex].

What is force?

A force is an effect that can change the motion of an object. A force can change the position of an object.

Force=mass*acceleration

How to calculate force?

We have been given mass of the object=808080 grams and acceleration =202020 meters per second.

We have to calculate force in kg per meters square so firstly we have to change mass which is in grams into kilogram=808080/1000=808.080 kg

Force =808.080*202020

=163248321

Force=163248321 kg [tex]m^{2}[/tex]

Hence is mass is 808080 grams and acceleration is 202020 meters per second then force will be 163248321 kg meters square.

Learn more about force at https://brainly.com/question/12970081

#SPJ2

Write 5 times x to the two thirds power in radical form.

Answers

(5 • x)^2/3

We know that the parentheses goes around 5 • x and that it also goes to the power of 2/3.

(17-7)•6+2+56-8
Pemdas

Answers

I don’t even know the answer to be honest
(10)x6+2+56-8
60+2+56-8
62+56-8
118-8
110

19 - 5 (2m - 3) + 9m = 36

Answers

Answer:

m = -2

Step-by-step explanation:

For this problem, we will simply solve the equation for m.

19 - 5 ( 2m - 3 ) + 9m = 36

19 - 10m + 15 + 9m = 36

34 - m = 36

-m = 2

m = -2

Hence, for this equation to be true, m must equal negative 2.

Cheers.

Other Questions
The place in a digital device where the data and programs that device is currently using are accessible by the processor is called the ___ Why is there an inherent conflict between planning and control uses of budgets? what did Europeans do with the gold and silver that was mine from Mexico to Peru If it's Monday at noon at the prime meridian, what is the time and day in;a. New York Cityb. San FranciscoC. Sydney, Australiad. Honolulu, HI A student is using a microscope to observe samples taken from pond water. She notices something that she thinks may be a living thing. Which feature would BEST support that the unknown sample is a living thing? Evaluate 1/3m11/2n when m=21 and n=12 2/3x+15=17 please and thank you an object is dropped from the top of a cliff 675 meters high. its height above the ground t seconds after it is dropped is 675-4.9t^2. determine its speed 7 seconds after it is dropped, the speed if the object 7 seconds after it's dropped in m/sec Why did the colonists pick that location for the Jamestown colony? Need help with this question. I tried simplifying the radical, but apparently its not the answer (Beetlejuice)what color is a carrot? How is the caliber of a rifle or handgun determined 12d^2 + 3dx + x- (-4d^2 + 2dx - 8x) A train which is traveling at 73mi/hr applies its brakes as it reaches point A and slows down with a constant deceleration. Its decreased velocity is observed to be 56mi/hr as it passes a point 1/2mi beyond A. A car moving at 45mi/hr passes point B at the same instant that the train reaches point A. In an unwise effort to beat the train to the crossing, the driver "steps on the gas." Calculate the constant acceleration a that the car must have in order to beat the train to the crossing by 3.9sec and find the velocity v of the car as it reaches the crossing. Describe the type of solution for the given system of linear equations.x + 4y = 78y = 14 - 20a. infinite solutionsb. two solutionsc. no solutiond. one solution what are the some difference between a formal work place and a informal work place I'm genuinely lost in any problems like this and my teachers are no help, I need help doing exponents in general, everything to do with them. I genuinely need help To combine integers with different signs find their _ _ then _ the smaller absolute value from the larger one give the sum the sign of the number with the _ absolute Compute $(1 + i)(2 + i)(3 + i).$ Simplify. 1/23(12+1)2 Enter your answer, as a simplified fraction, in the box.